6) The Axiom of Regularity - Suggested Solutions to Jech's Set Theory

$ \newcommand{\func}{\operatorname} \newcommand{\lcm}{\func{lcm}} \newcommand{\hl}[1]{\textbf{#1}} \newcommand{\lf}{\lfloor} \newcommand{\rf}{\rfloor} \newcommand{\f}[1]{\left\lf #1 \right\rf} \newcommand{\bb}[1]{\left( #1 \right)} \newcommand{\Sb}[1]{\left\lbrack #1 \right\rbrack} \renewcommand{\ss}[1]{\left\lbrace #1 \right\rbrace} \renewcommand{\mod}[1]{\left\lvert #1 \rightrvert} \newcommand{\R}{\boldsymbol{R}} \newcommand{\C}{\boldsymbol{C}} \newcommand{\Z}{\boldsymbol{Z}} \newcommand{\Q}{\boldsymbol{Q}} \renewcommand{\P}{\mathbb{P}} \newcommand{\Po}{\mathcal{P}} \newcommand{\F}{\mathcal{F}} \newcommand{\Zz}{\Z_{\geq 0}} \renewcommand{\c}[1]{\left\langle #1 \right\rangle} \renewcommand{\mod}[1]{\left\lvert #1 \right\rvert} \newcommand{\norm}[1]{\left\lVert #1 \right\rVert} \renewcommand{\bar}[1]{\overline{#1}} \newcommand{\eqbreak}{\phantom{{} = {}}} \newcommand{\impliesbreak}{\phantom{{} \implies {}}} \newcommand{\iffbreak}{\phantom{{} \iff {}}} \newcommand{\del}{\partial} \renewcommand{\d}[1]{\, \mathrm{d} #1} \newcommand{\dydt}{\frac{\d{y}}{\d{t}}} \newcommand{\dydx}{\frac{\d{y}}{\d{x}}} \newcommand{\pd}[2]{\frac{\del #1}{\del #2}} \newcommand{\pp}[3]{\frac{\del^2 #1}{\del #2 \del #3}} \newcommand{\dd}[2]{\frac{\mathrm{d} #1}{\mathrm{d} #2}} \newcommand{\Inn}{\func{Inn}} \newcommand{\Aut}{\func{Aut}} \newcommand{\diag}{\func{diag}} \newcommand{\id}{\func{id}} \newcommand{\GL}{\func{GL}} \newcommand{\SL}{\func{SL}} \newcommand{\End}{\func{End}} \newcommand{\Hom}{\func{Hom}} \newcommand{\Tr}{\func{Tr}} \newcommand{\rank}{\func{rank}} \newcommand{\ORD}{\mathbf{ORD}} \newcommand{\OD}{\mathbf{OD}} \newcommand{\HOD}{\mathbf{HOD}} \newcommand{\ZF}{\mathsf{ZF}} \newcommand{\ZFC}{\mathsf{ZFC}} \newcommand{\ZC}{\mathsf{ZC}} \renewcommand{\AC}{\mathsf{AC}} \newcommand{\DC}{\mathsf{DC}} \newcommand{\CC}{\mathsf{CC}} \newcommand{\CH}{\mathsf{CH}} \newcommand{\SH}{\mathsf{SH}} \newcommand{\KH}{\mathsf{KH}} \newcommand{\GCH}{\mathsf{GCH}} \newcommand{\PA}{\mathsf{PA}} \newcommand{\BST}{\mathsf{BST}} \newcommand{\MA}{\mathsf{MA}} \newcommand{\Con}{\func{Con}} \newcommand{\ON}{\mathbf{ON}} \newcommand{\dom}{\func{dom}} \newcommand{\ran}{\func{ran}} \newcommand{\pred}{\func{pred}} \newcommand{\mos}{\func{mos}} \newcommand{\WF}{\mathbf{WF}} \newcommand{\type}{\func{type}} \newcommand{\V}{\mathbf{V}} \renewcommand{\L}{\mathcal{L}} \newcommand{\cl}{\func{cl}} \newcommand{\trcl}{\func{tc}} \newcommand{\TC}{\func{TC}} \newcommand{\A}{\mathfrak{A}} \newcommand{\B}{\mathfrak{B}} \newcommand{\add}{\func{add}} \newcommand{\cov}{\func{cov}} \newcommand{\non}{\func{non}} \newcommand{\cf}{\func{cf}} \newcommand{\forces}{\Vdash} \newcommand{\one}{\mathbbm{1}} \newcommand{\restrictedto}{\mathord{\upharpoonright}} \newcommand{\Fn}{\func{Fn}} \newcommand{\Col}{\func{Col}} \newcommand{\bigtriangle}{\triangle} \newcommand{\U}{\mathcal{U}} \newcommand{\W}{\mathcal{W}} \newcommand{\N}{\mathcal{N}} \newcommand{\Seq}{\mathit{Seq}} \newcommand{\lh}{\func{lh}} \newcommand{\Ult}{\func{Ult}} \newcommand{\In}{\func{In}} \newcommand{\otp}{\func{otp}} \newcommand{\symdiff}{\, \triangle \,} \newcommand{\crit}{\func{crit}} \newcommand{\sat}{\func{sat}} \renewcommand{\int}{\func{int}} \newcommand{\length}{\func{length}} \newcommand{\Lim}{\func{Lim}} \newcommand{\Succ}{\func{Succ}} \newcommand{\G}{\mathcal{G}} \newcommand{\T}{\mathcal{T}} \newcommand{\ext}{\func{ext}} \newcommand{\height}{\func{height}} \renewcommand{\diamond}{\diamondsuit} \newcommand{\D}{\mathcal{D}} \newcommand{\NS}{\mathrm{NS}} $


Exercise 6.1.

Solution. We induct on $\rank(x)$. If $\rank(x) = 0$, then $\lbrace \rank(z) + 1 : z \in x\rbrace = \emptyset$, so $\sup\lbrace \rank(z) + 1 : z \in x\rbrace = 0$.

If $\rank(x) = \alpha + 1$, then $x \in V_{\alpha+1} \implies x \subseteq V_\alpha$. In other words, every $z \in x$ has $\rank(z) \leq \alpha$. Thus:
$$ \begin{align*}
\sup\lbrace \rank(z) + 1 : z \in x\rbrace \leq \sup\lbrace \alpha + 1 : z \in x\rbrace = \alpha + 1 = \rank(x)
\end{align*}$$ On the other hand, suppose $\beta := \sup\lbrace \rank(z) + 1 : z \in x\rbrace < \rank(x) \implies \sup\lbrace \rank(z) + 1 : z \in x\rbrace \leq \alpha$. Then $\beta + 1 \leq \alpha$, and $\rank(z) + 1 \leq \beta + 1 \implies z \in V_\beta$ for all $z \in x$. Thus, $x \subseteq P(V_\beta) = V_{\beta + 1} \subseteq V_\alpha$. contradicting the minimality of $\rank(x) = \alpha + 1$.

If $\rank(x) = \alpha$ is a limit ordinal, then we must have:
$$ \begin{align*}
\forall \beta < \alpha \, \exists z \in x \, (z \in V_\beta \wedge \forall \gamma < \beta \, (z \notin V_\gamma))
\end{align*}$$ In other words:
$$ \begin{align*}
\forall \beta < \alpha \, \exists z \in x \, (\rank(z) = \beta)
\end{align*}$$ which implies that $\sup\lbrace \rank(z) + 1 : z \in x\rbrace = \alpha$.

$\square$




Exercise 6.2.

Solution. We induct on $\alpha$ on the second assertion. For the base case $\alpha = 0$, we note that $V_\omega = \bigcup_{n < \omega} V_n$, and it's easy to see that $\vert V_{n+1}\vert = 2^{\vert V_n\vert }$. Thus, $V_\omega$ is a countable union of finite sets, hence:
$$ \begin{align*}
\vert V_\omega\vert \leq \sum_{n < \omega} \vert V_n\vert \leq \sum_{n < \omega} \aleph_0 = \aleph_0 \cdot \aleph_0 = \aleph_0
\end{align*}$$ On the other hand, clearly $\omega \subseteq V_\omega$, so $\vert V_\omega\vert \geq \aleph_0$.

If $\alpha = \beta + 1$, then $V_{\omega + \beta+1} = P(V_{\omega + \beta})$, so by Lemma 3.3 $\vert V_{\omega+\beta+1}\vert = 2^{\vert V_{\omega + \beta}\vert }$. By induction hypothesis, $\vert V_{\omega + \beta}\vert = \beth_{\omega + \beta}$, so $\vert V_{\omega+\beta+1}\vert = 2^{\beth_\beta} = \beth_{\omega+\beta+1}$.

If $\alpha$ is a limit ordinal, then first let:
$$ \begin{align*}
V_\beta' := V_\beta - \bigcup_{\gamma < \beta} V_\gamma
\end{align*}$$ Note that:
$$ \begin{align*}
\vert V_\beta'\vert =
\begin{cases}
\vert V_\beta\vert , &\text{if $\beta$ is not a limit ordinal} \\
0, &\text{otherwise}
\end{cases}
\end{align*}$$ and $\c{V_\beta' : \beta < \alpha}$ is a disjoint family of sets. Thus, we have:
$$ \begin{align*}
\vert V_{\omega+\alpha}\vert = \sum_{\beta < \omega + \alpha} \vert V_\beta'\vert = \vert \aleph_0 + \alpha\vert \cdot \sup_{\beta<\alpha} \beth_\beta = \vert \alpha\vert \cdot \beth_\alpha
\end{align*}$$ Since $\beth_\alpha \geq \alpha$ for all $\alpha$ (easy to prove via induction), we have that $\vert V_{\omega+\alpha}\vert = \beth_\alpha$.

$\square$




Exercise 6.3.


Lemma 6.3.A. If $\kappa$ is uncountable inaccessible, then $\kappa = \aleph_\kappa = \beth_\kappa$.

Proof. Note that clearly $\kappa \leq \aleph_\kappa \leq \beth_\kappa$. To show that $\beth_\kappa \leq \kappa$, since $\beth_\kappa = \sup\lbrace \beth_\alpha : \alpha < \kappa\rbrace$, it suffices to show that if $\alpha < \kappa$ then $\beth_\alpha < \kappa$.

We prove this by transfinite induction.
  1. Since $\kappa$ is uncountable, $\beth_0 = \aleph_0 < \kappa$.
  2. If $\beth_\alpha < \kappa$, then since $\kappa$ is a strong limit, we have that $\beth_{\alpha+1} = 2^{\beth_\alpha} < \kappa$.
  3. If $\alpha$ is a limit ordinal, then $\sup\lbrace \beth_\beta : \beta < \alpha\rbrace \leq \kappa$. Since $\kappa$ is regular and $\alpha < \kappa$, then inequality must be strict.

$\blacksquare$



Solution. If $\kappa = \omega$, then $\vert V_\omega\vert = \aleph_0$ by Exercise 6.2. Otherwise, since $\kappa$ is uncountable we have $\omega + \kappa = \kappa$ (here addition is ordinal addition). Thus, by Exercise 6.2 and Lemma 6.3.A, we have $\vert V_\kappa\vert = \beth_\kappa$.

$\square$




Exercise 6.4.

Solution. We can do better. We show that if $\rank(x) = \alpha$ and $\rank(y) = \beta$, then:
Set Rank
$\lbrace x,y\rbrace$ $\max\lbrace\alpha,\beta\rbrace + 1$
$\c{x,y}$ $\max\lbrace\alpha,\beta\rbrace + 2$
$x \cup y$ $\max\lbrace\alpha,\beta\rbrace$
$\bigcup x$ $\bigcup \alpha$
$P(x)$ $\alpha + 1$
$x^y$ $\max\lbrace\alpha,\beta\rbrace$
We use Exercise 6.1. It's clear that if $x \subseteq y$ then $\rank(x) \leq \rank(y)$.

$\lbrace x,y\rbrace$:
$$ \begin{align*}
\rank(\lbrace x,y\rbrace) &= \sup\lbrace \rank(x)+1,\rank(y)+1\rbrace \\
&= \max\lbrace \alpha+1,\beta+1\rbrace \\
&= \max\lbrace \alpha,\beta\rbrace + 1
\end{align*}$$ $\c{x,y}$:
$$ \begin{align*}
\rank(\c{x,y}) &= \rank(\lbrace \lbrace x\rbrace,\lbrace x,y\rbrace\rbrace) \\
&= \sup\lbrace \rank(\lbrace x\rbrace)+1,\rank(\lbrace x,y\rbrace)+1\rbrace \\
&= \max\lbrace \rank(x)+2,\rank(\lbrace x,y\rbrace)+1\rbrace \\
&= \max\lbrace \alpha+2,\max\lbrace \alpha,\beta\rbrace+2\rbrace \\
&= \max\lbrace \alpha,\beta\rbrace+2
\end{align*}$$ $x \cup y$:
$$ \begin{align*}
\rank(x \cup y) &= \sup\lbrace \rank(z) + 1 : z \in x \cup y\rbrace \\
&= \sup\lbrace \sup\lbrace \rank(z) + 1 : z \in x\rbrace, \sup\lbrace \rank(z) + 1 : z \in y\rbrace\rbrace \\
&= \sup\lbrace \rank(x),\rank(y)\rbrace \\
&= \max\lbrace \alpha,\beta\rbrace
\end{align*}$$ $\bigcup x$: We note that $z \in \bigcup x$ iff $z \in y$ for some $y \in x$. This implies that $\rank(z) + 2 \leq \rank(x)$, so $\rank(z) + 1 < \rank(x)$. Therefore:
$$ \begin{align*}
\rank\bb{\bigcup x} &= \sup\ss{\rank(z) + 1 : z \in \bigcup x} \\
&\leq \sup\lbrace \beta : \beta < \rank(x)\rbrace \\
&= \bigcup \alpha
\end{align*}$$ On the other hand, suppose $\rank\bb{\bigcup x} < \bigcup \alpha$, so there exists some $\beta < \alpha$ such that $\rank\bb{\bigcup x} < \beta$. Since $\rank(x) = \alpha$, there exists $y \in x$ such that $\rank(y) \geq \beta$. Since $\rank\bb{\bigcup x} < \beta$, this implies that $\rank(z) + 1 \geq \rank\bb{\bigcup x}$ for some $z \in y$. This contradicts that $\rank(z) + 1 < \rank(x)$.

$P(x)$: Since $x \in P(x)$, clearly $\rank(x) + 1 \leq P(x)$. On the other hand:
$$ \begin{align*}
\rank(P(x)) &= \sup\lbrace \rank(z) + 1 : z \in P(x)\rbrace \\
&= \sup\lbrace \rank(z) + 1 : z \subseteq x\rbrace \\
&\leq \sup\lbrace \rank(x) + 1 : z \subseteq x\rbrace \\
&= \rank(x) + 1
\end{align*}$$ $x^y$: We first note that for all $f \in x^y$, we have $f \subseteq y \times x$. Since:
$$ \begin{align*}
\rank(y \times x) &= \sup\lbrace \rank(\c{z,w}) + 1 : z \in y \wedge w \in x\rbrace \\
&= \sup\lbrace \max\lbrace \rank(z)+1,\rank(w)+1\rbrace + 1 : z \in y \wedge w \in x\rbrace \\
&\leq \max\lbrace \rank(y),\rank(x)\rbrace + 1 \\
&= \max\lbrace \alpha,\beta\rbrace + 1
\end{align*}$$ Therefore, $\rank(f) \leq \max\lbrace \alpha,\beta\rbrace + 1$ for all $f \in x^y$, so $\rank(x^y) \leq \max\lbrace \alpha,\beta\rbrace + 2$. On the other hand, for each $\gamma < \max\lbrace \alpha,\beta\rbrace$, let $\c{z,w} \in y \times x$ such that $\max\lbrace \rank(z),\rank(w)\rbrace > \gamma$. Then there exists a function $f \in x^y$ such that $\c{z,w} \in f$, Therefore:
$$ \begin{align*}
\rank(x^y) \geq \rank(f) + 1 \geq \max\lbrace \rank(z),\rank(w)\rbrace + 2 > \gamma + 2
\end{align*}$$ for all $\gamma < \max\lbrace \alpha,\beta\rbrace$.

$\square$




Exercise 6.5.

Solution. While these sets do not have universally agreed set-theoretic definitions, it is sufficient to define $\Z$, $\Q$ and $\R$ in $V_{\omega+\omega}$ in their "intended way".

Integers: For $\Z$, we may define it as (along with some operation $+$) the unique infinite cyclic group. we first note that $\boldsymbol{N} \in V_{\omega+1}$. Now define an equivalence relation $\sim$ on $\boldsymbol{N} \times \boldsymbol{N}$ by stipulating that:
$$ \begin{gather*}
(m,n) \sim (m',n') \\
\iff \\
\exists k \in \boldsymbol{N} \, ((m = n + k \wedge m' = n' + k) \vee (m + k = n \wedge m' + k = n))
\end{gather*}$$ Define $\Z := (\boldsymbol{N} \times \boldsymbol{N})/\sim$. Clearly $\Z$ can be defined in $V_{\omega+\omega}$. Now define the operation $+$ by stipulating that:
$$ \begin{align*}
[(m,n)] + [(m',n')] := [(m + m',n + n')]
\end{align*}$$ We first show that $+$ is well-defined (which will then clearly be definable in $V_{\omega+\omega}$). Suppose $(m_1,n_1) \sim (m_2,n_2)$ and $(m_1',n_1') \sim (m_2',n_2')$. Let $k,k' \in \boldsymbol{N}$, and for one case suppose they are such that $m_1 = n_1 + k$ and $m_1' = n_1' + k'$. Then:
$$ \begin{align*}
m_1 + m_1' = n_1 + n_1' + (k + k'), \; m_2 + m_2' = n_2 + n_2' + (k + k')
\end{align*}$$ The other three cases are similar. Clearly $+$ is commutative, associative, and $[(0,0)]$ the identity element. For inverse, define:
$$ \begin{align*}
[(m,n)]^{-1} := [(n,m)]
\end{align*}$$ Then $[(m,n)] + [(n,m)] = [(m+n,m+n)] = [(0,0)]$, so $(\Z,+)$ is the unique infinite cyclic group.

Rationals: For $\Q$, we consider another equivalence relation $\sim$ on $\Z \times (\Z - \lbrace 0\rbrace)$, where $0$ is the identity element of $(\Z,+)$, as follows (it is no longer relevant how $\Z$ is defined - it only matters that $\Z \in V_{\omega+\omega}$):
$$ \begin{align*}
(m,n) \sim (m',n') \iff \exists j,k \in \boldsymbol{N} \, (j \cdot m = k \cdot n \wedge j \cdot m' = k \cdot n')
\end{align*}$$ Here $\cdot$ denotes multiplication on $\Z$ which may be defined in the usual way using $+$. Now define $+,\times$ on $\Q/\sim$ by stipulating that:
$$ \begin{gather*}
[(m,n)] + [(m',n')] = (m \cdot n' + m' \cdot n,n \cdot n') \\
[(m,n)] \times [(m',n')] = (m \cdot m', n \cdot n')
\end{gather*}$$ One may show that $+$ and $\times$ are well-defined (and are clearly definable in $V_{\omega+\omega}$), and make $(\Q,+,\times)$ the commutative ring as intended.

Reals: For $\R$, they are canonically defined using Dedekind cuts on $\Q$ (see Theorem 4.4). All Dedekind cuts are clearly definable in $V_{\omega,\omega}$. Proving that the $+$ and $\times$ operations (which will be defined in the obvious sense) work are tedious - we refer to readers to Rudin's book.

$\square$




Exercise 6.6.


Exercise 6.6(i).

Solution. We induct on $\rank(x)$. Since $V_0 = \emptyset$, the base case is trivial. Suppose the statement holds for ranks $\beta < \alpha$, and suppose $\rank(x) = \alpha$.

$\implies$: If $x \in B$, then by definition we have $x \in A$ and $x \subseteq B \subseteq A$, i.e. $\forall y \in x \, (y \in B)$. Since $\rank(y) < \rank(x)$, we have that $\TC(y) \subseteq A$. Thus, $\TC(x) = x \cup \bigcup_{y \in x} \TC(y) \subseteq A$.

$\impliedby$: We note that if $y \in x$, then $\TC(y) \subseteq A$. By induction hypothesis, $y \in B$ for all $y \in x$, so $x \subseteq B$. Thus, $x \in B$.

$\square$




Exercise 6.6(ii).

Solution. Let $C \subseteq A$ be another transitive class. We induct on $\rank(x)$ showing that if $x \in C$, then $x \in B$. Again, base case is trivial, so suppose the statement holds for ranks $\beta < \alpha$, and suppose $\rank(x) = \alpha$.

Let $x \in C$. Since $C$ is transitive, $y \in C$ for all $y \in x$. Since $\rank(y) < \rank(x)$, by induction hypothesis we have that $y \in B$. In other words, $x \subseteq B$. By definition of $B$, $x \in B$.

$\square$